Let x , y , z , t be positive real numbers such that 1 + x 1 + 1 + y 1 + 1 + z 1 + 1 + t 1 ≥ 3 .
If the maximum value of x y z t can be expressed as q p , where p and q are coprime positive integers, then find p + q .
This section requires Javascript.
You are seeing this because something didn't load right. We suggest you, (a) try
refreshing the page, (b) enabling javascript if it is disabled on your browser and,
finally, (c)
loading the
non-javascript version of this page
. We're sorry about the hassle.
Nice solution! Never expect Jensen's inequality being used here!
Log in to reply
You must expect the unexpected ! ;)
Log in to reply
Yeah... I'm really impressed! :) Thanks for the lesson!
By the way, do you think you can solve this by only using Cauchy and/or AM - GM?
Log in to reply
@Steven Jim – I would think about doing it by AM-GM, you must change the part y x as b a or any two variables you like. It's confusing that the dummy variables coincide with the original variables
Log in to reply
@Aditya Narayan Sharma – Oh, really? I'm sorry if I made that mistake. I was a little bit sleepy when I posted the problem. Thanks for finding out!
Log in to reply
@Kushal Bose , Hey thanks for pointing out, I will modify that as soon as possible.
@Aditya Narayan Sharma Hey, check this one out!
Applying the AM-GM inequality, we have 1 + x 1 = ( 1 − 1 + y 1 ) + ( 1 − 1 + z 1 ) + ( 1 − 1 + t 1 ) = 1 + y y + 1 + z z + 1 + t t ≥ 3 3 ( 1 + y ) ( 1 + z ) ( 1 + t ) y z t Similarly, 1 + y 1 ≥ 3 3 ( 1 + x ) ( 1 + z ) ( 1 + t ) x z t ; 1 + z 1 ≥ 3 3 ( 1 + x ) ( 1 + y ) ( 1 + t ) x y t ; 1 + t 1 ≥ 3 3 ( 1 + x ) ( 1 + y ) ( 1 + z ) x y z
Multiple side by side all above inequalities, we get ( 1 + x ) ( 1 + y ) ( 1 + z ) ( 1 + t ) 1 ≥ ( 1 + x ) ( 1 + y ) ( 1 + z ) ( 1 + t ) 8 1 x y z t
Or x y z t ≤ 8 1 1 .
The equality holds iff x = y = z = t = 3 1 .
We can maximize this equation as we suppose WLOG, that x=y=z=t.
We observe that,
4/1+x >= 3
This gives us
x >= 1/3
So the value of xyzt = x^4
So, if we take the original equation, and raise both sides to the power of 4, we get.
x^4 >= 1/81 where g.c.d(1, 81) = 1
Hence, gives us the answer 1+81 = 82
Using symmetry arguments we can say that at maximum x y z t , x = y = z = t . This means x < = 3 1 . Hence maximum x y z t = 8 1 1 and hence the answer is 82.
Problem Loading...
Note Loading...
Set Loading...
Relevant wiki: Jensen's Inequality
Define f ( x ) = 1 − 1 + x 1 and notice that the given inequality is similar to c y c l i c ∑ ( 1 − 1 + x 1 ) ≤ 1 . Now since given quantities are positive we have f ′ ′ ( x ) > 0 which implies f is convex throughout the domain. Thus if we use jensen's inequality,
f ( x ) + f ( y ) + f ( z ) + f ( t ) ≥ 4 f ( 4 x + y + z + t ) = 4 − 1 + 4 x + y + z + t 4 , Now if α = x y z t then we can deduce by AM-GM that 4 x + y + z + t ≥ 4 α and thus − 1 + 4 x + y + z + t 1 ≥ − 4 α + 1 1 .
This hence reduces to 1 ≥ c y c l i c ∑ ( 1 − 1 + x 1 ) ≥ 4 − 1 + 4 α 4 , on solving we have α = x y z t ≤ 8 1 1 making the answer 8 2 .